Energie in einem elektrischen Feld unter einer Eichtransformation

Betrachten Sie eine (Test-) Ladung, die sich bei befindet R ( T ) in einem statischen elektrischen Feld mit Potential v ( R ) . Die Energie für dieses System ist gegeben durch

E = 1 2 M R ˙ ( T ) 2 + Q v ( R ( T ) ) .

(Man könnte ein Magnetfeld hinzufügen, ohne diese Energie zu verändern.) Es ist bekannt, dass das Potential durch eine Konstante verändert werden kann, ohne die Physik zu verändern; Wir können dies beheben, indem wir das Potenzial bei wählen Null sein (nehmen wir an, dass alle Felder zerfallen als R der Einfachheit halber).

Unter einer Spurweitentransformation haben wir

v v F T , A A + F .
Wenn ich wähle F um nur eine Funktion der Zeit zu sein, ist dies gleichbedeutend mit der Änderung des Potentials at auf zeitabhängige Weise, was im Wesentlichen einen zeitvariablen Offset zu dem obigen Energieausdruck hinzufügt. Wenn ich wähle F nur eine Funktion der Position zu sein, dann bleibt das elektrische Potential unverändert, und der Ausdruck für die Energie bleibt unverändert.

Meine Frage ist, wie man den Fall wo interpretiert F hängt sowohl von Raum als auch von Zeit ab? Unter einer solchen Eichtransformation wird die Energie des Systems

E = 1 2 M R ˙ ( T ) 2 + Q v ( R ( T ) ) Q F T | R ( T ) .
Wie ist dieser Begriff zu interpretieren? Warum ist die Energie bis auf eine Bezugspunktwahl nicht wohldefiniert? Während ich das durch die Anklage erzeugte Feld ignoriere, glaube ich nicht, dass ich es explizit berücksichtigen muss, um dies zu verstehen. Sicherlich im Testladungslimit dürfte es keine Auswirkungen haben. Beachten Sie, dass die Energie im Hintergrundfeld unter der Eichtransformation unveränderlich ist, da sie nur von den Integralen von E und B abhängt. Was vermisse ich?

Bearbeiten: Ich bin mir bewusst, dass der Hamiltonian für ein solches System nicht eichinvariant ist, obwohl die Bewegungsgleichungen es sind. Ich versuche zu verstehen, warum Energie keine "gute" Größe ist, um diese Situation zu beschreiben.

Antworten (1)

Das Problem ist, dass Sie irgendwie inkompatible Elemente des elektrostatischen und elektrodynamischen Kontexts miteinander kombinieren.

Wenn Sie in einem vollständig elektrostatischen Kontext arbeiten möchten, ist es am besten, immer im Coulomb-Eichmaß zu arbeiten. In diesem Fall wird die Energie eines geladenen Teilchens durch Ihre Formel angegeben. Aber in diesem Fall verlieren Sie Ihre Messfreiheit und können die von Ihnen beschriebene zeitabhängige Messtransformation nicht legitim durchführen.

Wenn Sie die volle Freiheit zur Durchführung von Eichtransformationen nutzen möchten, verwenden Sie implizit den vollständig dynamischen konzeptionellen Rahmen - selbst wenn Ihr spezifisches System zufällig statisch ist, ist die mathematische Möglichkeit, dass es dynamisch ist, in die Eichtheorie eingebaut Beschreibung. In diesem Fall müssen Sie vorsichtiger mit "Energie" umgehen. Die volle Erhaltungsgröße enthält Beiträge aus dem elektromagnetischen Feld. Selbst wenn Sie die EM-Felder festhalten (wie bei einer Testladung), müssen Sie aus Gründen der Konsistenz entweder den Hamilton- oder den Lagrange-Formalismus verwenden und nicht nur den einfachen Aufbau (kinetische + potentielle Energie) aus der Elementarmechanik. Das Vektorpotential A erscheint explizit sowohl im Hamiltonian als auch im Lagrangeian für ein geladenes Teilchen in einem externen Feld, und unter einer Eichtransformation transformiert es sich so, dass die Bewegungsgleichungen invariant bleiben.

Was die physikalische Interpretation betrifft – nun, sie ist notorisch düster. Ich kenne keine, und ich muss einfach der Mathematik folgen, wo immer sie mich hinführt.

Bearbeiten: Jeder Begriff der konservierten Energie stammt immer aus einer Art zeittranslationaler Invarianz der Physik, die die Zeitentwicklung eines Systems beschreibt, z. B. der Lagrangian oder Hamiltonian des Systems. Wenn das EM-Feld als dynamisch betrachtet wird und den Maxwell-Gleichungen gehorcht (die zeitlich translationsinvariant sind), erhalten Sie eine Energieerhaltung, die den Beitrag des EM-Felds enthält.

In Ihrem Fall geben Sie ein nicht dynamisches Hintergrund-EM-Feld an und berücksichtigen nur die Energie des geladenen Teilchens. Wenn sich das nicht-dynamische Hintergrund-EM-Feld im Laufe der Zeit ändert, gibt es im Allgemeinen überhaupt keine Vorstellung von Energieerhaltung, da das Teilchen Energie aus dem Hintergrundfeld gewinnen kann. In Ihrem Fall ist das EM-Feld elektrostatisch und zeitlich translationsinvariant. Diese zeittranslationale Invarianz kann jedoch durch ein Eichfeld "versteckt" werden, dessen Eichfreiheitsgrade sich mit der Zeit ändern. In diesem Fall ändert sich der Lagrangian/Hamiltonian oberflächlich mit der Zeit, was die formale Energieerhaltung durcheinander bringt. Damit die Energie erhalten bleibt, müssen Sie in einem Messgerät arbeiten, das die Zeit-Translations-Invarianz offensichtlich bewahrt (z. B. Coulomb-Eichgerät), damit Sie den Satz von Noether verwenden können. Q v ( R ) mit jeder Art von Teilchen potentieller Energie.

Auch wenn ich zur Hamiltonschen Formulierung gehe und die Hamiltonsche Verwendung schreibe ( P Q A ) 2 / 2 M anstatt M v 2 / 2 Um Bewegung in einem externen Feld zu beschreiben, ist der Hamilton-Operator aufgrund dieses zusätzlichen Terms aus dem elektrischen Potential unter einer Eichtransformation nicht invariant. Beachten Sie, dass ( P Q A ) 2 / 2 M IS-Eicheninvariante. Das ist einfach das Problem, dass der Hamiltonoperator für ein Teilchen in einem externen Feld nicht eichinvariant ist (siehe z. B. physical.stackexchange.com/questions/94699/… ). Die Bewegungsgleichungen sind natürlich egal.
Vielleicht kann ich meine Frage anders formulieren: Warum ist die Energie für ein Teilchen, das sich in einem externen EM-Feld bewegt, nicht genau definiert? Ist es gut definiert, wenn ich alle Quellen in mein System einbeziehe? Was ist die entscheidende Zutat, die zu diesem Unterschied führt?
@Jolyon Bearbeitet.
Ok, gut genug für meine Zufriedenheit :-)